Đến nội dung

Hình ảnh

BĐT AM-GM


  • Please log in to reply
Chủ đề này có 339 trả lời

#141
no matter what

no matter what

    Why not me

  • Thành viên
  • 397 Bài viết

23,Chứng mi9nh với mọi a,b,c dương có tổng bình phương bằng 1
$a+b+c+\frac{1}{abc}\geq 4\sqrt{3}$. Tổng bình phương =1 ??????
giả sử a là min(a,b,c)
do b,c<1 do đó ta đánh giá : $a\leq \frac{1}{9}=>\frac{1}{abc}\geq \frac{1}{a}\geq 9>4\sqrt{3}$
ko biết đúng ko.

rất tiếc đây là 1 lời giải sai chú ý là với $a=b=c=\frac{1}{\sqrt{3}}$,BĐT trên đúng

Hic làm ko ra mình hoá rồ , Xin lỗi bạn...
Bài 21: áp dụng :$$a^3+1\leq (\frac{a^2+2}{2})^2$$ ta chứng minh:
$$\sum 4\frac{a^2}{(a^2+2)(b^2+2)}\geq \frac{4}{3}\Leftrightarrow 2(a^2+b^2+c^2)+(ab)^2+(cb)^2+(ac)^2\geq 72$$
bất đẳng thức cuối đúng theo AM_GM

Đây là offical solution của APMO 2005,è 1 vài cách giải khác cho bài này,mình xin dc post sau(ngủ cái đã :wacko: )

#142
duong vi tuan

duong vi tuan

    Thượng sĩ

  • Thành viên
  • 229 Bài viết

rất tiếc đây là 1 lời giải sai chú ý là với $a=b=c=\frac{1}{\sqrt{3}}$,BĐT trên đúng


Đây là offical solution của APMO 2005,è 1 vài cách giải khác cho bài này,mình xin dc post sau(ngủ cái đã :wacko: )


Sặc sao hôm nay pro lên nhanh vậy , đang ăn cơm chợt nhớ lại cái lời giảI mình mới đăng .... :( :( :( . Bài đó cũng đơn giản nhưng do nhầm lẫn ngu ngốc ( căn 3= 9 :wacko: :wacko: :wacko: )
$a+b+c+\frac{1}{9abc}\geq \frac{4}{\sqrt{3}}$
và $abc\leq \frac{1}{3\sqrt{3}}$ . :wacko: :wacko: :wacko:
NGU
Hình đã gửi

#143
duong vi tuan

duong vi tuan

    Thượng sĩ

  • Thành viên
  • 229 Bài viết
25,Chứng minh với mọi a,b,c dương ta có
$a^3+b^3+c^3\geq 2abc+\frac{a^2+b^2+c^2}{\frac{1}{a}+\frac{1}{b}+\frac{1}{c}}$
bất đẳng thức đương đương: $$(a^3+b^3+c^3)(\frac{1}{a}+\frac{1}{b}+\frac{1}{c})=(a+b+c)^2$$ đúng theo bunhia .
bài 26, cho a=b=c=1 vào .... ( lần này chắc đúng :icon6: )
30) mình dùng S_V ( do bài này nổi tiếng với S-V mà :ohmy: ):$$6\sum \sqrt{\frac{2a}{a+b}}=\sum \sqrt{\frac{9(ab+ac)}{(a+b+c)}\frac{8(a+b+c)}{(b+c)(a+b)}}\leq \sqrt{\frac{18\sum ab}{a+b+c}.16\frac{(a+b+c)^2}{\prod (a+b)}}\leq 3$$
ở trên sử dụng bdt cơ bản $9\prod (a+b)\geq 8(\sum a)(\sum ab)$

32,Cứng mnih với mọi a,b,c dương
$\frac{1}{a}+\frac{1}{b}+\frac{1}{c}\geq \frac{4a}{2a^2+b^2+c^2}+\frac{4b}{2b^2+c^2+a^2}+\frac{4c}{2c^2+a^2+b^2}$
ta có:$\frac{4a}{2a^2+b^2+c^2}\leq \frac{4a}{2ab+2ac}=\frac{2}{b+c}\leq \frac{1}{2}(\frac{1}{b}+\frac{1}{c})$
tương tự ta có dpcm

34,Chứng minh với mọi a,b có tổng bằng 1
$(1+\frac{1}{a})(1+\frac{1}{b})\geq 9$

Bất đẳng thức đã cho tương đương:$\frac{1}{ab}\geq 4$
cho x=2 , y=-1 , ko đúng ( chả biết sao nữa :wacko: :wacko: )


40,Chứng minh với mọi a,b,c dương
$\frac{a+b+c}{3}\leq \frac{1}{4}\sqrt[3]{\frac{\left [ (a+b)(b+c)(c+a) ^2\right ]}{abc}}$

áp dụng : $9\prod (a+b)\geq 8(\sum a)(\sum ab)$
ta có $$\frac{[(a+b)(b+c)(c+a)]^2}{abc}\geq \frac{8^2}{9^2}\frac{[(a+b+c)(ab+bc+ca)]^2}{abc}\geq (\frac{4}{3})^2(a+b+c)^3$$

Bài viết đã được chỉnh sửa nội dung bởi duong vi tuan: 09-01-2013 - 14:27

NGU
Hình đã gửi

#144
Dung Dang Do

Dung Dang Do

    Dũng Dang Dở

  • Thành viên
  • 524 Bài viết
Góp vui tý :(chế đó) :)
$\boxed{\text{Bài 41}}$
Cho $x,y,z>0$ CMR:
+$$\frac{(\frac{1}{x}+\frac{2}{\sqrt{y}}+\frac{3}{\sqrt[3]{z}})(x^2+y+\sqrt[3]{z}^2)}{36}\ge \sqrt[6]{xyz}$$

Bài viết đã được chỉnh sửa nội dung bởi Dung Dang Do: 12-01-2013 - 13:21

@@@@@@@@@@@@

#145
Nguyen Tho The Cuong

Nguyen Tho The Cuong

    Hạ sĩ

  • Thành viên
  • 53 Bài viết
cho x thoả mãn $0\leq x\leq 1$. Tìm MAX :

$13\sqrt{x^{2}-x^{4}}+9\sqrt{x^{2}+x^{4}}$

#146
Oral1020

Oral1020

    Thịnh To Tướng

  • Thành viên
  • 1225 Bài viết

cho x thoả mãn $0\leq x\leq 1$. Tìm MAX :

$13\sqrt{x^{2}-x^{4}}+9\sqrt{x^{2}+x^{4}}$

http://boxmath.vn/4rum/f22/toan-10-chung-minh-bat-dang-thuc-23170/
:D

"If I feel unhappy,I do mathematics to become happy.


If I feel happy,I do mathematics to keep happy."

Alfréd Rényi

Hình đã gửi


#147
Oral1020

Oral1020

    Thịnh To Tướng

  • Thành viên
  • 1225 Bài viết

Góp vui tý :(chế đó) :)
$\boxed{\text{Bài 41}}$
Cho $x,y,z>0$ CMR:
+$$\frac{(\frac{1}{x}+\frac{2}{\sqrt{y}}+\frac{3}{\sqrt[3]{z}})(x^2+y+\sqrt[3]{z}^2)}{36}\ge \sqrt[6]{xyz}$$

Bất đẳng thức này sai với $x=y=z=0,2013$ :D

"If I feel unhappy,I do mathematics to become happy.


If I feel happy,I do mathematics to keep happy."

Alfréd Rényi

Hình đã gửi


#148
nguyencuong123

nguyencuong123

    Thiếu úy

  • Thành viên
  • 587 Bài viết
Làm giúp em bài này: Cho a,b,c $>$0 , Thoả mãn: $a^{2}+b^{2}+c^{2}=1$
Chứng Minh :
$\frac{a}{b^{2}+c^{2}}+\frac{b}{c^{2}+a^{2}}+\frac{c}{a^{2}+b^{2}}\geq \frac{3\sqrt{3}}{2}$

    :icon12:  :icon12:  :icon12:   Bình minh tắt nắng trời vương vấn :icon12:  :icon12:  :icon12:       

      :icon12: Một cõi chơi vơi, ta với ta  :icon12:       

:nav: My Facebook  :nav:  

 


#149
no matter what

no matter what

    Why not me

  • Thành viên
  • 397 Bài viết

Làm giúp em bài này: Cho a,b,c $>$0 , Thoả mãn: $a^{2}+b^{2}+c^{2}=1$
Chứng Minh :
$\frac{a}{b^{2}+c^{2}}+\frac{b}{c^{2}+a^{2}}+\frac{c}{a^{2}+b^{2}}\geq \frac{3\sqrt{3}}{2}$

Trang 1 bạn ơi,kĩ thuật đánh giá mẫu ấy

#150
N H Tu prince

N H Tu prince

    Sĩ quan

  • Thành viên
  • 388 Bài viết

Làm giúp em bài này: Cho a,b,c $>$0 , Thoả mãn: $a^{2}+b^{2}+c^{2}=1$
Chứng Minh :
$\frac{a}{b^{2}+c^{2}}+\frac{b}{c^{2}+a^{2}}+\frac{c}{a^{2}+b^{2}}\geq \frac{3\sqrt{3}}{2}$

Trang 1 bạn ơi,kĩ thuật đánh giá mẫu ấy

Nếu sử dụng kĩ thuật đánh giá mẫu $VT\ge \frac{a+b+c}{2}\ge \frac{3\sqrt{3}}{2}??????$
Nếu sử dụng C-S thì dễ rồi,còn muốn giải theo AM-GM thì khá ảo
ta CM BDT phụ $1-a^2\le \frac{2\sqrt{3}}{9a}$ bằng cách vận dụng AM-GM
$(\frac{\sqrt{3}-1}{2})(\frac{\sqrt{3}+1}{2})(1-a)(1+a).a\le [\frac{(\frac{\sqrt{3}-1}{2}).(1+a)+(\frac{\sqrt{3}+1}{2}).(1-a)+a}{3}]^3=\frac{\sqrt{3}}{9}=>1-a^2\le \frac{2\sqrt{3}}{9a}=>\frac{a}{1-a^2}\ge \frac{9a^2}{2\sqrt{3}}$
Xây dựng các BDT tương tự,cộng lại ta có $Q.E.D$

Bài viết đã được chỉnh sửa nội dung bởi hoangngocbao1997: 15-01-2013 - 21:52

Link

 


#151
no matter what

no matter what

    Why not me

  • Thành viên
  • 397 Bài viết

Nếu sử dụng kĩ thuật đánh giá mẫu $VT\ge \frac{a+b+c}{2}\ge \frac{3\sqrt{3}}{2}??????$
Nếu sử dụng C-S thì dễ rồi,còn muốn giải theo AM-GM thì khá ảo
ta CM BDT phụ $1-a^2\le \frac{2\sqrt{3}}{9a}$ bằng cách vận dụng AM-GM
$(\frac{\sqrt{3}-1}{2})(\frac{\sqrt{3}+1}{2})(1-a)(1+a).a\le [\frac{(\frac{\sqrt{3}-1}{2}).(1+a)+(\frac{\sqrt{3}+1}{2}).(1-a)+a}{3}]^3=\frac{\sqrt{3}}{9}=>1-a^2\le \frac{2\sqrt{3}}{9a}=>\frac{a}{1-a^2}\ge \frac{9a^2}{2\sqrt{3}}$
Xây dựng các BDT tương tự,cộng lại ta có $Q.E.D$

Bạn xem lại trang 1 (http://diendantoanho...-gm/page__st__0) ở kĩ thuật đánh giá mẫu+BĐT phụ ấy ,bài này là 1 VD của mình mà,thâmk chí còn có với 4 biến nữa(hoàn toàn giai bằng AM-GM ) :icon6:

Bài viết đã được chỉnh sửa nội dung bởi no matter what: 15-01-2013 - 22:05


#152
duc12116

duc12116

    Trung sĩ

  • Thành viên
  • 108 Bài viết

Góp vui tý :(chế đó) :)
$\boxed{\text{Bài 41}}$
Cho $x,y,z>0$ CMR:
+$$\frac{(\frac{1}{x}+\frac{2}{\sqrt{y}}+\frac{3}{\sqrt[3]{z}})(x^2+y+\sqrt[3]{z}^2)}{36}\ge \sqrt[6]{xyz}$$

Cái anh Oral1020 nói nhảm. Khi x=1,y=2,z=1/3 dấu = vẫn xảy ra mà. Để đó, dăm ba ngày nữa tôi sẽ giải ra!!!

#153
Oral1020

Oral1020

    Thịnh To Tướng

  • Thành viên
  • 1225 Bài viết

Cái anh Oral1020 nói nhảm. Khi x=1,y=2,z=1/3 dấu = vẫn xảy ra mà. Để đó, dăm ba ngày nữa tôi sẽ giải ra!!!

Cái đó thì dấu $=$ thôi.chứ thử thay số khác thì ngược dấu(Chủ bài cũng đã nói với mình là đề có lổi)

"If I feel unhappy,I do mathematics to become happy.


If I feel happy,I do mathematics to keep happy."

Alfréd Rényi

Hình đã gửi


#154
duc12116

duc12116

    Trung sĩ

  • Thành viên
  • 108 Bài viết
Công nhận thế thật. Khi thay cặp số, chẳng hạn 16,36,27 thì BĐT đúng, nhưng nếu chọn 1,2,3 thì BĐT lại đổi chiều.
Trông bài toán đẹp thế mà lại sai, tiếc thật

#155
nguyencuong123

nguyencuong123

    Thiếu úy

  • Thành viên
  • 587 Bài viết

Cho các số thực không âm a,b,c thoả mãn

$a^{2}+b^{2}+c^{2}=3$

Chứng minh rằng :

$\frac{a}{a^{2}+2b+3}+\frac{b}{b^{2}+2c+3}+\frac{c}{c^{2}+2a+3}\leq \frac{1}{2}$

    :icon12:  :icon12:  :icon12:   Bình minh tắt nắng trời vương vấn :icon12:  :icon12:  :icon12:       

      :icon12: Một cõi chơi vơi, ta với ta  :icon12:       

:nav: My Facebook  :nav:  

 


#156
no matter how

no matter how

    Binh nhì

  • Thành viên
  • 17 Bài viết

Cho các số thực không âm a,b,c thoả mãn

$a^{2}+b^{2}+c^{2}=3$

Chứng minh rằng :

$\frac{a}{a^{2}+2b+3}+\frac{b}{b^{2}+2c+3}+\frac{c}{c^{2}+2a+3}\leq \frac{1}{2}$

giải cho nó có phong trong cái :Dùng kí hiệu $\sum$ cho tắt nhé
$\sum \frac{a}{a^2+2b+3}\leq \sum \frac{a}{2(a+b+1)}$
ta chỉ cần CM $\sum \frac{a}{a+b+1}\leq 1$$hay \sum \frac{b+1}{a+b+1}\geq 2$
THeo C-S $\sum \frac{b+1}{a+b+1}= \sum \frac{(b+1)^2}{(b+1)(a+b+1)}\geq \frac{(\sum a+3)^2}{\sum (b+1)(a+b+1)}=2$(do $a^2+b^2+c^2=3)
P/s:nguồn bầiày ở đâu thế nhỉ,lammf thấy quen quá

#157
duong vi tuan

duong vi tuan

    Thượng sĩ

  • Thành viên
  • 229 Bài viết

làm bài này nhé
Cho a,b,c$\geq$0vaf a+b+c=3. CM
$\sum \frac{a^{2}}{a+2b^{2}}\geq 1$

$\sum \frac{a^{2}}{a+2b^{2}}\geq\sum a-\sum \frac{2ab^2}{a+2b^2}\geq 3-\frac{2}{3}\sum \sqrt[3]{a^2b^2}\geq 3-\frac{2}{3}\sum \frac{ab+ab+1}{3}=\frac{7}{3} - (\frac{4}{9}\sum ab) \geq 1$
lời giải trên dùng cô si ngược.

Bài viết đã được chỉnh sửa nội dung bởi duong vi tuan: 02-03-2013 - 08:22

NGU
Hình đã gửi

#158
duong vi tuan

duong vi tuan

    Thượng sĩ

  • Thành viên
  • 229 Bài viết

Thôi thì xấu tốt gì cũng xin trình bày thêm 1 cách nữa cho bài 16
$(\frac{x}{y}+\frac{z}{\sqrt[3]{xyz}})^2\geq (\frac{x}{y}+\frac{3z}{x+y+z})^2\geq (2\sqrt{\frac{3xz}{y(x+y+z)}})^2=12.\frac{xz}{y(x+y+z)}$
Xây dựng các BĐT còn lại là xong :icon6:

Bài :19,Chứng minh với mọi a,b,c dương ta có
$\frac{a^2}{a^2+bc}+\frac{b^2}{b^2+ca}+\frac{c^2}{c^2+ab}\leq \frac{a+b+c}{2\sqrt[3]{abc}}$



Mình xin post lời giải cua 1 bạn tên Hưng :lol: :lol: :lol:
Trước hết ta cm bổ đề sau:$(a+b+c)^4\geq \frac{27}{2}(a^2b^2+b^2c^2+c^2a^2+abc(a+b+c))$
các bạn có thể cm nó = cô si .
bất đang thức 19 tương đương: $\sum \frac{bc}{a^2+bc}+\frac{a+b+c}{2\sqrt[3]{abc}}\geq 3$
ta có
$\sum \frac{bc}{a^2+bc}\geq \frac{(ab+bc+ca)^2}{a^2b^2+b^2c^2+c^2a^2+abc(a+b+c)}=1+\frac{abc(a+b+c)}{a^2b^2+b^2c^2+c^2a^2+abc(a+b+c)}\geq 1+\frac{27abc(a+b+c)}{2(a+b+c)^4}=1+\frac{27abc}{2(a+b+c)^3}$
tu đó: $VT\geq 1+ \frac{27(abc)}{2(a+b+c)^3}+\frac{a+b+c}{6\sqrt[3]{abc}}+\frac{a+b+c}{6\sqrt[3]{abc}}+\frac{a+b+c}{6\sqrt[3]{abc}}\geq 3$ theo bdt cô si
NGU
Hình đã gửi

#159
Atu

Atu

    Hạ sĩ

  • Thành viên
  • 53 Bài viết

Cho a,b,c là các số thực ko âm t/m $a^{2}+b^{2}+c^{2}=1$. Tìm max:
$P=\left ( a+b+c \right )^{3}+a(2bc-1)+b(2ac-1)+c(2ab-1)$


Ta có:
$(a+b+c)^{3}+\sum a(2bc-1)$
$=(a+b+c)^{3}+6abc-(a+b+c)$
$=(a+b+c)(\sum a^{2}+2\sum ab-1)+6abc$
$=2(a+b+c)(\sum ab)+6abc$
Áp dụng bđt AM-GM ta có:
$2(\sum a)(\sum ab)\leq 2\sum a\leq 2\sqrt{3\sum a^{2}}=2\sqrt{3}$
$1=\sum a^{2}\geq 3\sqrt[3]{(abc)^{2}}\Rightarrow (abc)^{2}\leq \frac{1}{27}\Rightarrow 6abc\leq \frac{2}{\sqrt{3}}$
Cộng vế theo vế, ta có $max$ P =$\frac{8\sqrt{3}}{3}$

#160
dinhthanhhung

dinhthanhhung

    Trung sĩ

  • Thành viên
  • 111 Bài viết

Cho a,b,c là các số thực ko âm t/m $a^{2}+b^{2}+c^{2}=1$. Tìm max:
$P=\left ( a+b+c \right )^{3}+a(2bc-1)+b(2ac-1)+c(2ab-1)$

Bài này đơn giản quá :D
$P=(a+b+c)\left [(a+b+c)^{2}-1 \right ]+8abc\leq \sqrt{3(\sum a^{2})}(3\sum a^{2}-1)+\frac{8}{3}(\sum a^{2}+\sqrt{3\sum a^{2}})$




2 người đang xem chủ đề

0 thành viên, 2 khách, 0 thành viên ẩn danh